LSAT and Law School Admissions Forum

Get expert LSAT preparation and law school admissions advice from PowerScore Test Preparation.

User avatar
 Dave Killoran
PowerScore Staff
  • PowerScore Staff
  • Posts: 5849
  • Joined: Mar 25, 2011
|
#44058
Complete Question Explanation
(The complete setup for this game can be found here: lsat/viewtopic.php?t=16041)

The correct answer choice is (D)

Answer choice (A) is incorrect because G is selected but K is not.

Answer choice (B) is incorrect because J is selected but M is not.

Answer choice (C) is incorrect because only one tenant is selected, a violation of the first rule.

Answer choice (E) is incorrect because both M and P are selected, a violation of the last rule.

Thus, answer choice (D) is the correct answer.

Get the most out of your LSAT Prep Plus subscription.

Analyze and track your performance with our Testing and Analytics Package.